- Sun Jan 20, 2013 12:00 am
#25637
Complete Question Explanation
(The complete setup for this game can be found here: lsat/viewtopic.php?t=7800)
The correct answer choice is (A)
If one of the two visits to M is third, the other visit cannot be second or fourth in compliance with
the third and fifth rules:
Upon further inspection, you should realize that the range of possible solutions is wide open. This
is because the M > G > M sequence can be satisfied in weeks 1-2-3, but also in weeks 3-4-5 and
3-5-6. Rather than create three local diagrams, focus on the question at hand: when asked which
variables could be fourth and fifth, respectively, check to see which variables cannot be fourth
and fifth. In other words, proceed by the process of elimination:
From the first rule, we know that J cannot be visited fourth, eliminating answer choice (B). And,
since no destination is visited in two consecutive weeks, M cannot be visited fourth. This easily
eliminates answer choice (C).
Answer choice (D) cannot be true, because visiting J requires visiting G (not T) immediately
before J. While T could be fourth and J could be fifth in two different solutions, together they
create a TJ Block, which is impossible to satisfy.
Answer choice (E) is also impossible, because it creates an MTM Block in positions 3-4-5, which
violates the third rule.
With four of the answer choices eliminated, answer choice (A) is the only viable contender. For
the sake of efficiency, you need not verify that G and T could be fourth and fifth, respectively:
this answer choice must be correct, as none of the other answers provided an accurate list of
variables that can occupy these two spaces together. Here, this solution is tested for pedagogical
purposes only:
G and T are fourth and fifth, respectively, then J must be visited second to allow for the
creation of a GJ Block. Also, M must be visited sixth in compliance with the M > G > M
sequence established by the third rule. Thus, we arrive at a viable solution to the game:
(The complete setup for this game can be found here: lsat/viewtopic.php?t=7800)
The correct answer choice is (A)
If one of the two visits to M is third, the other visit cannot be second or fourth in compliance with
the third and fifth rules:
Upon further inspection, you should realize that the range of possible solutions is wide open. This
is because the M > G > M sequence can be satisfied in weeks 1-2-3, but also in weeks 3-4-5 and
3-5-6. Rather than create three local diagrams, focus on the question at hand: when asked which
variables could be fourth and fifth, respectively, check to see which variables cannot be fourth
and fifth. In other words, proceed by the process of elimination:
From the first rule, we know that J cannot be visited fourth, eliminating answer choice (B). And,
since no destination is visited in two consecutive weeks, M cannot be visited fourth. This easily
eliminates answer choice (C).
Answer choice (D) cannot be true, because visiting J requires visiting G (not T) immediately
before J. While T could be fourth and J could be fifth in two different solutions, together they
create a TJ Block, which is impossible to satisfy.
Answer choice (E) is also impossible, because it creates an MTM Block in positions 3-4-5, which
violates the third rule.
With four of the answer choices eliminated, answer choice (A) is the only viable contender. For
the sake of efficiency, you need not verify that G and T could be fourth and fifth, respectively:
this answer choice must be correct, as none of the other answers provided an accurate list of
variables that can occupy these two spaces together. Here, this solution is tested for pedagogical
purposes only:
G and T are fourth and fifth, respectively, then J must be visited second to allow for the
creation of a GJ Block. Also, M must be visited sixth in compliance with the M > G > M
sequence established by the third rule. Thus, we arrive at a viable solution to the game:
You do not have the required permissions to view the files attached to this post.